2
$\begingroup$

We say that an $n\times n$ matrix $A_n$ is a random symmetric Bernoulli matrix if each entry $a_{ij}$ is $\pm 1$ with probability $1/2$, the entries $a_{ij}$ with $i\ge j$ are independent and $a_{ij}=a_{ji}$. I would like a reference (and ideally an elementary proof) of the following fact: $$\text{var}(\lambda_1(A_n))\sim n^{-1/3}$$

I would also be happy with an upper bound $\text{var}(\lambda_1(A_n))\le f(n)$ with $\displaystyle\lim_{n\rightarrow \infty} f(n)=0$ if the proof is simple enough.

$\endgroup$
4
  • 1
    $\begingroup$ "of the following fact" -- Why is this a fact? $\endgroup$ Commented Feb 20 at 18:52
  • $\begingroup$ See remark 2.6 on page 18 of van Handel's lecture notes: web.math.princeton.edu/~rvan/APC550.pdf $\endgroup$ Commented Feb 20 at 19:07
  • $\begingroup$ I see no proof or reference there. $\endgroup$ Commented Feb 20 at 19:36
  • $\begingroup$ The paper [arxiv.org/pdf/2207.00546] shows that the fluctuations of the largest eigenvalue of $H := \frac{1}{\sqrt{n}}A_n$ around $2$ are of order $O(n^{-2/3})$, so the variance of order $O(n^{2(-2/3+1/2)} = n^{-1/3})$ is believable. $\endgroup$ Commented Feb 20 at 19:48

1 Answer 1

3
$\begingroup$

I believe this result follows from Corollary 15 in the paper of Tao and Vu, Random Matrices: sharp concentration of eigenvalues, Random Matrices Theory Appl. 2 (2013), no. 3, 1350007, 31 pp.

$\endgroup$

You must log in to answer this question.

Start asking to get answers

Find the answer to your question by asking.

Ask question

Explore related questions

See similar questions with these tags.